Difference between revisions of "2004 AMC 12A Problems/Problem 6"

(New page: ==Problem== Let <math>U=2\cdot 2004^{2005}</math>, <math>V=2004^{2005}</math>, <math>W=2003\cdot 2004^{2004}</math>, <math>X=2\cdot 2004^{2004}</math>, <math>Y=2004^{2004}</math> and <math...)
 
Line 18: Line 18:
  
 
==See Also==
 
==See Also==
 +
{{AMC12 box|year=2004|ab=A|num-b=5|num-a=7}}

Revision as of 11:07, 5 December 2007

Problem

Let $U=2\cdot 2004^{2005}$, $V=2004^{2005}$, $W=2003\cdot 2004^{2004}$, $X=2\cdot 2004^{2004}$, $Y=2004^{2004}$ and $Z=2004^{2003}$. Which of the following is the largest?

$\mathrm {(A)} U-V \qquad \mathrm {(B)} V-W \qquad \mathrm {(C)} W-X \qquad \mathrm {(D)} X-Y \qquad \mathrm {(E)} Y-Z \qquad$

Solution

$U-V=2004^{2005}$

$V-W=2004^{2004}$

$W-X=2001*2004^{2004}$

$X-Y=2004^{2004}$

$Y-Z=2003*2004^{2003}$

After comparison, $U-V$ is the largest. $\mathrm {(A)}$

See Also

2004 AMC 12A (ProblemsAnswer KeyResources)
Preceded by
Problem 5
Followed by
Problem 7
1 2 3 4 5 6 7 8 9 10 11 12 13 14 15 16 17 18 19 20 21 22 23 24 25
All AMC 12 Problems and Solutions